The PTA prepared 10 gallons of punch to serve at their first meeting. If each serving is eight ounces, how many people can be served?

Answers

Answer 1

Based on mathematical operations, the number of people that 10 gallons of punch can serve is 192 persons.

How is the number determined?

The number of people can be determined using mathematical operations of multiplication based on converting the gallons to ounces and division.

The number of gallons of punch to serve at the first meeting = 10 gallons

The quantity of punch per serving = 8 ounces

1 gallon = 153.722 ounces

10 gallons = 1537.22 ounces

The persons that can be served 8 ounces from the 10 gallons = 192 (1,537.22/8).

Thus, mathematically, from the 10 gallons of punch, the PTA can serve 192 persons.

Learn more about mathematical operations at https://brainly.com/question/20628271

#SPJ1


Related Questions

Can someone help me please

Answers

The next correct step of the following elimination method is (C) subtract to eliminate 'x'

What is the elimination method?The elimination method involves taking one variable out of the system of linear equations by using addition or subtraction along with multiplication or division of the variable coefficients. The problem is reduced to solving a single variable equation using the elimination method. Finding the values of x and y without changing the equations is fairly challenging. The x s cancel out and the x is removed from the equations when they are added together. As a result, it is known as the "elimination method."

So, the next correct step of the elimination method:

We can see that in both equations, '-3x' is the same and this means we will cancel the x variable.

Then, option (A) and option (D) is incorrect.

We know that (+ -) is always (-) but (- -) is always (+).And we need to make if -3x and +3x to cancel the 'x' variable.

Thus, the correct option will be an option (C).

Therefore, the next correct step of the following elimination method is (C) subtract to eliminate 'x'

Know more about the elimination method here:

https://brainly.com/question/25427192

#SPJ13

Kelsey has an orange ribbon and a blue ribbon. The orange ribbon is 7 11/12 inches long and the blue ribbon is 17 5/12 inches long. How much longer is the blue ribbon than the orange ribbon?

Answers

The blue ribbon is 2.2 times longer than the orange ribbon

How to determine the ratio

From the information given, we have that;

The orange ribbon is 7 11/12 inches long The blue ribbon is 17 5/12 inches long

Now, lets convert the mixed fractions to improper fractions;

Length of orange ribbon = 95/12

Length of blue ribbon = 209/12

The ratio is expressed as;

Length of blue ribbon/length of orange ribbon

Substitute the values, we have;

= 209/12 ÷ 95/12

Take the inverse of the divisor and multiply

= 209/ 12 × 12/95

= 2. 2 times

Hence, it is 2.2 times longer.

Learn more about ratio here:

https://brainly.com/question/2784798

#SPJ1

PLEASE HELP ME!!! due tonight *will make brainlist , if answer is correct*

Answers

i think it’s y=3x+8 not sure though

Calculator
What is the area of a rectangle with vertices at
ter your answer in the box.
(0,-4).(-1, -3) (2, 0), and (3, − 1)?

Answers

Answer:

The answer to your question is: 6 units

Step-by-step explanation:

Data

A (-4, 0)

B (-3,1)

C (0,-2)

D (-1,-3)

Formula

d = √((x2 - x1)² + (y2 - y1)²)

Process

dAB = √((x2 - x1)² + (y2 - y1)²)                 dAC = √((0 + 4)² + (-2 + 0)²)

      = √((-3 + 4)² + (1 - 0)²)                      dAC = √((4)² + (-2)²)

      = √((1)² + (1)²)                                    dAC = √(16 + 4)

      = √1 + 1                                             dAC = √20

dAB = √2  = dCD

dAD = √((-1 + 4)² + (-3 - 0)²)                    

dAD = √((3)² + (-3)²)

dAD = √9 + 9

dAD = √18 = dBC

reactangle's area = base x height = √18 x √2 = √18 x 2 = √36

                                                       = 6 units

The answer is six units


Which expression is equivalent to 2.5 + 3a
5.5a - 29.5
3a - 32
3a - 27
5.5a - 10.5
Mak
20 9.5?

Answers

Answer:d

Step-by-step explanation:

which is a appropriate estimate for 53,320x599

Answers

The answer after its estimation will be 3,00,00,000.

Here, the 53,320 will be estimated to 50,000 and 599 will be estimated to 600.

So, on multiplying, we get;

50,000×600 = 3,00,00,000

To offer an estimated, projected result rather than an exact result, we employ approximations in our calculations. Calculations are easier to handle when they are estimated using round numbers.

The result of rounding 109 to the closest ten is 110. and 100 is the nearest hundred. 2) Round the number downward if it is followed by the digits 0, 1, 2, 3, or 4. 102, for instance, is 100 when rounded to the closest ten.

To know more about estimation refer to the given link:

https://brainly.com/question/14806778?referrer=searchResults

#SPJ1

Create a division equation that represents the question: How many \frac{3}{8} are in \frac{5}{4} ?

Answers

The division equation that represents number of 3/8 in 5/4 is 10/3 = x.

Given,

Fractions : 3/8 and 5/4

We have to find a division equation for number of 3/8 in 5/4

Number of 3/8 in 5/4 = x

Now,

Number of a in b = b/a

So here,

Number of 3/8 in 5/4, x = 5/4 / 3/8

Number of 3/8 in 5/4, x = 5/ 3/2

Number of 3/8 in 5/4, x = 5 × 2 / 3

Number of 3/8 in 5/4, x = 10/3

That is,

The division equation that represents number of 3/8 in 5/4 is 10/3 = x.

Learn more about division equation here:

https://brainly.com/question/12066883

#SPJ1

How do you write 8.82 x 102 in standard form?​

Answers

Answer: Multiply 102 by 8.82 = 899.64x

PLEASE HELP I AM GROUNDED AND MY MOTHER IS SO MAD AT ME FOR MY GRADE

Answers

Answer: b=116, c=64

Step-by-step explanation:

to collect these data, the researchers randomly selected american households to visit and interviewed the adult member of the household whose birthday was nearest. is this an experiment or an observational study?

Answers

In an observational study, we measure or survey members of a sample without trying to affect them. In an experiment, we assign people or things to groups and apply some treatment to one of the groups, while the other group does not receive the treatment.

This is clearly an observational study

Learn more about observational study:

brainly.com/question/17269327

#SPJ4

A cookie factory uses 1/3 of a barrel of oatmeal in each batch of cookies. The factory used 2/3 of a barrel of oatmeal yesterday. How many batches of cookies did the factory make?

Answers

Answer:

it would be 89

Step-by-step explanation:

Find the measure of each acute angle in a right triangle in which the measure of one acute angle is 11 times the measure of the other acute angle.

Answers

The measure of the acute angles in the triangle are 7.5 and 82.5

How to determine the measure of the acute angles?

From the question, we have the following parameters:

One acute angle is 11 times the measure of the other acute angle.

Let the acute angles be x and y

Such that

y = 11x

The sum of the acute angles in a right triangle is 90

So, we have

x + y = 90

This gives

x + 11x = 90

So, we have

12x = 90

Divide by 12

x = 7.5

Recall that y = 11x

So, we have

y = 11 x 7.5

Evaluate

y = 82.5

Hence, the acute angles are 7.5 and 82.5

Read more about acute angles at

https://brainly.com/question/17118374

#SPJ1

a professor wants to study the effectiveness of a new study tool for a course. there are 150150150 students registered for the course. the professor ranks the students according to cumulative average in the prerequisite course. for every 222 students, in order, from the list, the professor flips a coin to assign one student to use the new study tool and the other to use the previous study tool. what type of experiment design is this?

Answers

It is a completely randomized design.

What is completely randomized design ?

Completely randomized designs are used in experiment design to explore the effects of a single primary factor without taking additional nuisance variables into account. This article describes single-factor, entirely randomized designs. On the basis of the various levels of that primary factor, the experiment compares the values of a response variable. The levels of the main factor are given at random to the experimental units in fully randomized designs.

Objects or subjects are assigned to groups in a completely random design at random. Labeling each subject before choosing from the labeled participants in a table of random numbers is one common practice for allocating subjects to treatment groups. Computers can be used to accomplish this as well.

To learn more about completely randomized design click here:

https://brainly.com/question/16024549

#SPJ4

please help like soon

Answers

The distance between the two points is 10 units and the slope of the line is -78/25

Distance Between Two Points

To find the distance between two points, we can simply use the formula of distance between two which is given as

[tex]d=\sqrt{(y_2-y_1)^2 + (x_2 - x_1)^2}[/tex]

Let's substitute the values and solve for the distance.

[tex]d=\sqrt{(y_2-y_1)^2 + (x_2 - x_1)^2}\\d = \sqrt{(1-7)^2 + (-3 - 5)^2} \\d = 10[/tex]

2)

The slope of the line can be found using the two points given.

[tex]m = \frac{y_2 - y_1}{x_2 - x_1} \\[/tex]

let's substitute the values and solve.

[tex]m = \frac{y_2 - y_1}{x_2 - x_1} \\\\m = -\frac{78}{25}[/tex]

Learn more distance between two points here;

https://brainly.com/question/7243416

#SPJ1

Measure of angle 1 = 110 degrees. You only need to type 110.

Answers

The unknown angles are as follows:

∠1 = 110°∠2 = 70°∠3 = 110°∠4 = 70°∠5 = 110°∠6 = 70°∠7 = 110°∠8 = 70°How to find angles?

When parallel lines are cut by a transversal line, angle relationships are formed such as corresponding angles, linear angles, alternate angles etc.

Therefore,

∠1 = 110 degrees

∠2 = 180 - 110 (sum of angles on a straight line)

∠2 = 70°

∠3 = ∠1 (vertically opposite angles)

Vertically opposite angles are congruent. vertically opposite angles share the same vertex

Therefore,

∠3 = 110°

∠2 = ∠4 (vertically opposite angles)

∠4 = 70°

∠1 = ∠5 (corresponding angles)

Corresponding angles are congruent

∠5 = 110°

∠6 = ∠4(alternate angles)

Alternate angles are congruent.

∠6 = 70°

∠1 = ∠7 (alternate exterior angles)

∠7 = 110°

∠2 = ∠8(alternate exterior angles)

∠8 = 70°

learn more on angles here: brainly.com/question/18166236

#SPJ1

Jocelyn has a points card for a movie theater.
. She receives 25 rewards points just for signing up.
• She earns 12.5 points for each visit to the movie theater.
. She needs at least 170 points for a free movie ticket.
Write and solve an inequality which can be used to determine v, the number of visits
Jocelyn can make to earn her first free movie ticket. Inequality?

Answers

Answer: 25 + 12.5v ≥ 170

Step-by-step explanation:

We already have the 25 reward points; it also gives the information that she gets 12.5 points for each visit. Because we don't know the amount she has gone to the theater or will go we have to use a variable; 12.5v. When we add them, we get 25 + 12.5v. The inequality we use is ≥ because it says "at least". When this is used that means it has to be equal to or greater and that is why use that sign and get the full equation;

5 + 12.5v ≥ 170

Answer:

55

Step-by-step explanation:

What is the equation of the line that is perpendicular to y = 2x + 3 and passes through the point (−4, 8). pls show work

Answers

Answer:

y = -1/2x + 6.

Step-by-step explanation:

The line perpendicular to y = 2x + 3 has slope of - 1/2 so we can write it as

y = -1/2x + c

When x = -4, y = 8 so we substitute:

8 = -1/2 (-4) + c

8 = 2 + c

c = 8-2 = 6

So, the answer is

y = -1/2x + 6.

Answer:

y = -1/2x + 6.

Step-by-step explanation:

5/6a-3=5/8

How would you solve this?

Answers

Step-by-step explanation:

[tex] \frac{5}{6} (a) - 3 = \frac{5}{8} \\ lcd = 48 \\ \\ \frac{5}{6} (a)(48) - 3(48) = \frac{5}{8} (48) \\ 40a - 144 = 30 \\ 40a = 30 + 144 \\ 40a = 174 \\ \frac{40a}{40} = \frac{174}{40} \\ a = 4\frac{14}{40} \\ a = 4 \frac{7}{20} [/tex]

I FIRST FOUND THE LCD AFTER THAT MULTIPLY EACH AND EVERY TERM IN THE EQUATION BY IT TO HET RID OF THE FRACTIONS AND SOLVE FOR a.

NEED HELP ASAP WILL MARK BRAINLIEST

Answers

Answer:

b -3

Step-by-step explanation:

t-3(4-t)=-24 multiply

t-12+3t=-24 simplify

4t-12=-24 plus 12 on both side

4t=-12 divide 4 on both side

t=-3

Jason bought four tickets to a basket ball game. He paid a total of $42.10 for the tickets and his parking was $6.50. What is the cost of each ticket

Answers

The cost of each ticket is 8.9 dollars.

What is cost price?

In mathematics, the term cost price can be explained with the help of profit which states, as the amount which is on the commodity and in the form of standard price called cost price. For the profit, the amount of the commodity can be designed by the shopkeeper which is bigger amount as per the cost price.

According to the question, the Jason bought 4 tickets for basket ball game. He paid a total cost of $42.10 for the tickets and for parking as $6.50

The paid amount for the 4 tickets is: $42.10.

And the parking cost is: $6.50

Total cost price without parking is: $42.10 - $6.50 = $35.60

Therefore, to calculate the cost of each ticket is: 35.60/4 = 8.9 dollars

Hence, the cost of each ticket is 8.9 dollars.

To learn more about the cost price from the given link:

brainly.com/question/1078746

#SPJ9

You start with $100 at a casino and can bet any amount of that money to play a game any amount of times where in each game, you have a 40% chance of winning. If you win, you’ll gain the amount you bet, in addition to being returned the amount you bet. If you lose, you lose your bet. What is your strategy to have the highest chance of getting to $200? (100 - 200 words)

Answers

Our strategy to win $200 is to use $100 in just one chance. It is the best bet for winning $200.

Given that we start with $100 at a casino and can bet any amount of that particular money to play a game to any amount of times where we have a 40% chance of winning in each game. If we win, we will gain the amount we bet and return the amount that we bet. If we lose, we will lose our bet.

It has been asked that what should be our strategy to win get $200.

We can write from the given information that:

Probability of wining = 40 % = 0.4

Probability of losing = 1 - 0.4 = 0.6

The best bet to win $200 is to bet $100 in one go itself.

For the above bet chance of winning $100 is 0.4

But let us assume that we are betting $50 two times,

Then L-Lose and W-Win

There will be 4 rational possible cases (WW,WL,LW,LL)

Probability of all the 4 above given cases will be as follows:

Probability of (WW) = 0.4 X 0.4 = 0.16

Probaility of (WL) = 0.4 X 0.6 = 0.24

Probability of (LW) = 0.6 X 0.4 = 0.24

Probability of (LL) = 0.6 X 0.6 = 0.36

Now for winning $100 we need to win both the bets of $50.

But the probability of winning both the bets = 0.16 ( very less than 0.4)

Hence the best bet would be to use $100 in one chance itself.

To know more about bet click here:

https://brainly.com/question/28813795

#SPJ4

Our strategy to win $200 is to use $100 in just one chance. It is the best bet for winning $200.

It has been asked that what should be our strategy to win get $200.

We can write from the given information that:

Probability of wining = 40 % = 0.4

Probability of losing = 1 - 0.4 = 0.6

The best bet to win $200 is to bet $100 in one go itself.

For the above bet chance of winning $100 is 0.4

But let us assume that we are betting $50 two times,

Then L-Lose and W-Win

There will be 4 rational possible cases (WW,WL,LW,LL)

Probability of all the 4 above given cases will be as follows:

Probability of (WW) = 0.4 X 0.4 = 0.16

Probability of (WL) = 0.4 X 0.6 = 0.24

Probability of (LW) = 0.6 X 0.4 = 0.24

Probability of (LL) = 0.6 X 0.6 = 0.36

Now for winning $100 we need to win both the bets of $50.

But the probability of winning both the bets = 0.16 ( very less than 0.4)

Hence the best bet would be to use $100 in one chance itself.

learn more about of probability here:

https://brainly.com/question/11234923

#SPJ4

The graph shows y varying directly as x according to the equation y = kx. What is the value of d?
A d = kb + kdd = kb + kd
B d = ka + kcd = ka + kc
C d = b + kdd = b + kd
D d = kb

Answers

The equation that represents the value of d is d = b + kc

How to determine the value of d?

The graph that completes the question is added as an attachment

The equation of the graph is given as

y = kx

Because the graph shows a direct variation, the equation can be illustrated as

y = mx

Where m represents the slope of the line

By comparing the equation y = mx and y = kx, we can conclude that

m = k

This means that the slope of the equation y = kx is k

From the graph, we have the points

(x, y) = (a, b) and (a + c, d)

The slope of a line is calculated using

Slope = (y₂ - y₁)/(x₂ - x₁)

Where

Slope = k

(x₁, y₁) = (a, b) and (x₂, y₂) = (a + c, d)

So, we have

k = (d - b)/(a + c - a)

Evaluate the like terms

k = (d - b)/(c)

Multiply both sides by c

d - b = kc

Add b to both sides

d = b + kc

Hence, the value of d is d = b + kc

Read more about linear equations at

https://brainly.com/question/4074386

#SPJ1

nathan is making a beaded bracelet. for every 5 red beads there are 3 white beads. what is the ratio of red to white beads?

Answers

Answer:

5;3

Step-by-step explanation:

Juanita wants to buy a computer. She needs to earn at least 400 dollars. If she makes 9 dollarsan hour and has already saved 40 dollars, how many hours does she need to work? Write an inequality and solve.

Answers

Problem

Juanita wants to buy a computer. She needs to earn at least 400 dollars. If she makes 9 dollars an hour and has already saved 40 dollars, how many hours does she need to work?

Solution

Let x the number of hours required we can set up the following inequality:

40 + 9x > 400

And now we can solve for x we can subtract 40 and we got:

9x > 360

And then we got:

x> 40

So then Juanita needs to work at least 40 hours

And the graph of the inequality is given by:

Evaluate the following expressions picture attached 100 POINTS

Answers

Answer:

3 5/14 (second choice)

Step-by-step explanation:

c = -3 1/7

d = 6.5 (6 1/2 as fraction)

-3 1/7 as improper fraction: -22/7

6 1/2 as improper fraction: 13/2

Now put both fractions into a common denominator

-22/7 = -44/14

13/2 = 91/14

Now add:

-44/14 + 91/14 = 47/14

Change 47/14 back to fraction

47/14 = 3 5/14

Please help!!
I’ll give you the Brainiest

You are shopping for T-shirts. Store A has T-shirts
priced at $25 for 2 shirts. Store B has T-shirts
priced at $42 for 4 shirts. Which store offers the
better deal?

Answers

Answer: The answer is the $42 T-Shirts

Step-by-step explanation: 42 divided by 4 is 10.5 dollars per t-shirt.

25 divided by 2 would mean 12.5 dollars per t shirt. There is only a 2 dollar difference, but it is clear that the 4 t-shirt deal is the best.

How many pounds of apples must chris pick before Pam’s orchard is cheaper that david’s

Answers

The total pounds of apples that Chris must pick before Pam’s orchard is cheaper that David's is 14.

How many apples should Chris pick?

The first step is to determine the total cost of picking apples from both orchards.

Cost of picking apples from David's Orchard : (10 x $2) + [(x - 10) x $1]

$20 + $x - $10

= $10 + $x

Cost of picking apples from Pam's Orchard : $10 + ($10 x $1.5) + [(x - 10) x 0.75]

$10 + $15 + 0.75x - 7.5

= $17.50 + 0.75x

When  Pam's Orchard is cheaper, the equation becomes :

$10 + $x  > $17.50 + 0.75x

$x - 0.75x > $17.50 - $10

0.25x > $7.50

x > $7.50 / 2.5

x > 3

Total pounds should be greater than 3, so it can be 4.

Total pounds = 10 + 3 = 14

To learn more about inequality, please check: https://brainly.com/question/5031619

#SPJ1

The diagram below shows the first four of a series of designs that an artist draws on artwork.

The artist wants to use a design with a maximum of 15 shaded circles. What number of design should the artist use?

Answers

Using an arithmetic sequence, the number of design that the artist should use is:

b. Design 6.

What is an arithmetic sequence?

In an arithmetic sequence, the difference between consecutive terms is constant and is called common difference d.

The nth term of an arithmetic sequence is given by the rule presented as follows:

[tex]a_n = a_1 + (n - 1)d[/tex]

In which [tex]a_1[/tex] is the first term of the sequence.

The sequence for the number of circles is:

(4, 6, 8, ...)

Hence the common difference is given by:

d = 8 - 6 = 6 - 4 = 2.

Then the rule for the nth term is:

[tex]a_n = 4 + 2(n - 1)[/tex]

As the first term is of 4.

The maximum number is of 15 shaded circles, hence:

4 + 2(n - 1) ≤ 15

4 + 2n - 2 ≤ 15

2 + 2n ≤ 15

2n ≤ 13

x ≤ 6.5.

Hence the maximum number is of 6, meaning that Design 6 is chosen, given by option b.

Missing information

The complete problem is given by the image at the end of the answer.

More can be learned about arithmetic sequences at https://brainly.com/question/6561461

#SPJ1

A new club sent out 152 coupons to boost sales for next year's memberships. They provided 3 times as many to potential members than to existing members. How many coupons did they send to existing members?

Answers

The club has to send 57 coupons to existing members.

Given,

The number of coupons sent out by a new club to boost sales for next year's membership = 152

They provide 3 times as many to potential members than to existing members

We have to find the number of coupons they send to existing members;

Here,

x be the number of coupons to existing members.

Then,

3x = 152

Now,

x = 152/3 = 50.66 ≈ 57

That is,

The club has to send 57 coupons to existing members.

Learn more about number of coupons here;

https://brainly.com/question/28736307

#SPJ1


XYZ and LMN are complementary angles. XYZ = (3x+14)° and LMN= (5x+24)°. What is LMN in degrees?
Write your answer as a decimal.

Answers

Answer:

6.5°

Step-by-step explanation:

We know that both XYZ and LMN are complementary angles.

Complementary angles add up to 90°.

So,

we can simply add both of the expressions and set it equal to 90 (degrees).

(3x+14) + (5x+24) = 90

-> add the variables and constants

8x+38=90

-> subtract the 38 from both sides (90-38)

8x=52

-> divide out the coefficient to isolate the variable

x=6.5

So, the answer is 6.5 degrees.

Hope this helped :)

Other Questions
A person with clinical depression will experience at least four signs of the disorder for _____. at least two weeks at least two days more than five days a full day What does Odysseus think that the Cyclops should give him because he is his guest?1. A meal2. A gift3. A sheep4. A bed Match the word with its definition.Question 4 options:EmpresarioMestizoAnglo-American Filibuster1. a white person living in the United States who is not hispanic2. a person of mixed Spanish and Native American heritage3. a land agent whose job it was to bring new settlers into an area4. an individual who carries out unauthorized warfare against a foreign country Graph the dilated image of triangle XYZ using a scale factor of 1.5 and (0,0) as the center of dilation b.This interval includes one endpoint but not the other; do you see why?We can express this range of values in inequality notation and in interval notation: Inequality notation: -4 -4 AND 2 4) Interval notation: (-4,4)Rewrite each of the following inequalities in interval notation:a.-8 -7-6-8 -7-6 -5 -4-8 -7 -6 -5 -4 -3-8 -7 -6-5 -4-B -7 -6-5C.d.0.-5 -4 -3 -2 -10123as an inequality: -7 Solve quickly!3(x+10)=4(x-2) a florist shop gets an average of 18 phone orders in a 2 hour time period. in order to find the probability that the florist shop will get at most 4 phone orders in a 30 minute period using the poisson distribution, what is the average number of phone orders per 30 minutes? There were 18,652 geese on a lake. What is this number rounded to the tenthousands place?A20,000B19.600C19.000D18.600car mechanic rounds a car's weight to the nearest thousand. He says theweight is 4,000 pounds. What is the least and the greatest amounts the carcould actually weigh? Explain. A researcher studying the nutritional value of a new candy places a 5.30 g sample of the candy inside a bomb calorimeter and combusts it in excess oxygen. The observed temperature increase is 2.49 C. If the heat capacity of the calorimeter is 40.80 kJK1, how many nutritional Calories are there per gram of the candy Geometry help for A and B. Answer B approximate volume? What are some outcomes of the townshend acts? the nurse reviews the physician's emergency department progress notes for the client who sustained a head injury and sees that the physician observed the battle sign. the nurse knows that the physician observed which clinical manifestation? You are catering at a ceremony and have a budget of R1 200 to spend on meat. Youwant to serve chicken and steak. Chicken costs R30 per kilogram and steak costs R120per kilogram.3.1.1 In the problem description in 3.1 above, state what changes and what remainsthe same. In other words, mention the variables for the problem. You and your wife (travel) around the world Let f(x)=(x7)(x+6)(x5). Find the y-intercept(s), the x-intercept(s), the values of x where f(x)>0, and the values of x where f(x) whats 1 plus 1 in mathmaticals Jim, Eduardo, Larry, Simone, Ian, Dawn, Tyrone, and Kim have all been invited to a dinner party. They arrive randomly and each person arrives at a different time.a. In how many ways can they arrive?b. In how many ways can Jim arrive first and Kim Last?c. Find the probability that Jim will arrive first and Kim last. Joe has extremely shaky and unsteady hands due to a minor medical condition. This can be a tricky condition to deal with because Joe is also a photographer. What tool might be helpful for Joe to overcome this issue?Joe has extremely shaky and unsteady hands due to a minor medical condition. This can be a tricky condition to deal with because Joe is also a photographer. What tool might be helpful for Joe to overcome this issue? Paul has brown eyes, a dominant trait, and his wife Emily has blue e trait. Paul and Emily's daughter has blue eyes. How is this possible? Merlyns weight went down from 125 pounds to 110 pounds after dieting. What was her percent weight loss?